You are on page 1of 17

1.

Invariante
1.1. Introduccin
"...El profesor dice: -Haremos un experimento. Escriban, por favor, en sus cuader-
nos 11 nmeros, 6 ceros y 5 unos. Ahora, efecten 10 veces seguidas la siguiente
operacin: tachen dos cualesquiera nmeros, y si ellos eran iguales, terminen de
escribir a los nmeros restantes un cero, y si son diferentes -un uno. Empiezen,
por favor. Terminaron? Y ahora les voy a decir que nmero ustedes tienen.
Todos ustedes debieron obtener un uno! '
Esta escena provoca la siguiente pregunta natural: De dnde el profesor sabe
de ante mano la respuesta que obtuvieron los estudiantes?
Es que, las operaciones se pueden realizar de diferentes mtodos. El asunto
es, que despus de cada operacin, la suma de todos los nmeros en los cuadernos
ser impar necesariamente como al principio. Verificar esto es fcil, cada vez la
suma o no cambia o cambia por 2. Esto significa, que despus de 10 operaciones,
el nmero restante debe ser impar, osea igual al . Al explicar esto, el profesor
seguramente pronuncia la palabra mgica "invariante".
Qu es esto? La palabra "invariante" significa algo que no vara, como en
nuestro ejemplo no variaba la paridad de las cifras.
Aqu tenemos otro ejemplo del invariante.
Problema 1.
En el alfabeto de la tribu UAU hay slo dos letras: U y A. Adems esta
lengua posee las siguientes propiedades. Si de la palabra quitamos U A (es decir,
las letras U y A que estn juntas), entonces el significado de la palabra no vara. El
sentido de la palabra tampoco vara al adicionar en cualquier lugar de la palabra
la combinacin de letras AU o UUAA. Se puede afirmar, que las palabras UAA
y AUU tienen el mismo significado?
Solucin 1.
Observen que para cualquier operacin de adicin o de supresin del segmento
de la palabra, la cantidad de letras U y A en ese segmento es igual. Esto significa,
que la diferencia de las cantidades de las letras U y A en la palabra no vara, es
decir, es constante. Observen esto en el ejemplo:
A -* AAU -> AUUAAAU - AUAAU.
En todas estas palabras la cantidad de letras A es una ms, que la cantidad
de letras U. Regresemos a la solucin. En la palabra UAA la diferencia es igual a
(1), y en la palabra AUU es igual a 1. Esto significa, que de la palabra UAA no se
Figure 1.1:
puede obtener la palabra AUU con las operaciones permitidas y por consiguiente,
no se puede afirmar que estas palabras tienen el mismo significado necesariamente.
Esta solucin ilustra la idea principal del uso del invariante. Nos han dado
algunos objetos, con los cuales nos permiten realizar unas operaciones determi-
nadas. Despus de esto surge la siguiente pregunta. Se puede obtener un objeto
de otro, utilizando estas operaciones? Para responder a esta pregunta construimos
alguna magnitud, la cual no cambia con las operaciones indicadas. Si el valor de
esta magnitud no es igual para dos objetos indicados, entonces, es cierto que la
respuesta a la pregunta formulada es negativa.
Analicemos un otro ejemplo mas:
Problema 2.
Un crculo est dividido en 6 sectores (ver fig.1.1 (1)), en cada uno de los cuales
hay una ficha. Est permitido mover cualesquiera dos fichas a los sectores vecinos
en una jugada . Se podr reunir todas las fichas en un sector con la ayuda de
tales operaciones?
Solucin 2.
Enumeramos los sectores con nmeros de 1 hasta 6 en sentido de las manecillas
del reloj (ver fig.1.1 (2)). Para cualquier posicin de las fichas consideramos la
siguiente magnitud: S es la suma de los nmeros de los sectores, en los cuales
estn las fichas dadas (teniendo en cuenta la multiplicidad).
Ejemplo. Para la posicin en la fig.1.1 (3) tenemos S 2+2+4+4+5+6 = 23.
Est claro, que para el desplazamiento de una ficha a el sector vecino el
sumando, correspondiente a ella en la suma S , cambia la paridad. Esto sig-
nifica que si se desplazan simultneamente dos fichas, entonces la paridad de la
magnitud S no vara, ella es una invariante. Pero para la posicin en la fig.l
S 21. Si todas las fichas se encuentran en un sector con el nmero ^ 4, entonces
S 6^4. es un nmero par (y el nmero 21 es impar). Por consiguiente, de la
posicin inicial no se puede obtener la posicin, en la cual todas las 6 fichas se
encuentran en un sector (ver la fig.3).
A veces el invariante se usa no slo para demostrar que algn objeto no se
puede obtener del dado, sino para averiguar cuales objetos se pueden obtener del
objeto inicial. Esto ilustra el siguiente problema.
Problema 3.
En el tablero hay escritos los nmeros 1,2,3,..., 19,20. Se permite borrar
cualesquiera dos nmeros a y fe, y en vez de ellos escribir el nmero a + b 1.
Qu nmero quedar despus de 19 de tales operaciones?
Solucin 3.
Para cualquier conjunto de n nmeros en el tablero Xi,x<,...,x
n
considere-
mos la siguiente magnitud X (x\ + 3 + +
x
n) ~
n
- Supongamos que con el
conjunto se realiza la operacin descrita en el problema. Como vara X ? S la
suma de todos los nmeros del conjunto, menos a y 6, es igual a 5, entonces antes
de la transformacin de la magnitud X era igual a S + a + b n, y despus de
la transformacin X S + (a + b 1) (n 1) S + a + b n. As, el valor
de la magnitud X no vari, ella es un invariante. Inicialmente (para el conjunto
de la condicin del problema) X = (1 + 2 + . . . + 19' + 20) - 20 = 190. Esto
significa que despus de 19 operaciones, cuando en el tablero quede un nmero
p, X tambin ser igual a 190. Pero por su definicin, en ese momento X ser
igual a p 1. Significa que p 191. Por consiguiente, el nmero, que queda en el
tablero necesaiamente ser igual a 191.
Para los profesores. S usted escucha en la clase la solucin de este prob-
lema, entonces lo ms seguro es, que este se oir de la siguiente forma: en cada
paso la suma de todos los nmeros disminuye en 1. Los pasos son 19, y la suma
inici amiente era igual 210. Esto significa que al fin y al cabo la suma es igual a
210 19.= 191. Aunque esta solucin es cierta, hay que explicar a los estudiantes
que este problema pertenece al tema "invariante". El asunto es que en el caso
dado, el invariante es tan simple que admite una interpretacin completamente
trivial. En el problema siguiente tal "reduccin" no sale bien, aunque sea parecido
al problema 3.

^k
Hg. 4
Figure 1.2:
Problema 4.
En el tablero hay escritos los nmeros 1, 2 , . . . , 20. Est permitido borrar dos
cualesquiera nmeros a y 6, y cambiarlos por el nmero ab + a + b. Qu nmero
puede quedar en el tablero despus de 19 de tales operaciones?
Sugerencia.
En calidad de invariante consideren la siguiente magnitud; el producto de
todos los nmeros incrementados previamente en uno antes de la multiplicacin.
Aqu tenemos varios problemasnotables.
Problema 5.
Sobre 6 palmeras hay 6 loros, en cada palmera hay un loro. Las palmeras
crecen en fila con intervalos de 10 metros. Si un loro vuela de una palmera a
otra, entonces un otro loro necesariamente vuela la misma distancia pero en otra
direccin. Pueden todos los loros reunirse en un palmera? Y s hay 7 loros y 7
palmeras?
Problema 6.
En una tabla de dimensin 8x8, una de las casillas (cuadrculas) est pintada
de color negro y todas las restantes estn pintadas de color blanco. Demuestre
que no se pueden obtener todas las casillas blancas con la ayuda de las siguientes
operaciones sobre las filas y las columnas: en una fila o en una columna todas las
casillas cambian el color por su inverso?
Problema 7.
Resuelvan el mismo problema para una tabla de dimensin 3x3 (ver la fig.1.2).
Como antes slo una casilla (de la esquina) est pintada de color negro.
Problema 8.
Resuelvan el mismo problema para una tabla de 8 x 8, si todas las cuatro
casillas de las esquinas estn pintadas de color negro,
Problema 9.
En el tablero hay escritos los nmeros 1, 2, 3, . . . , 1989. Se permite borrar dos
cualesquiera nmeros y escribir en vez de ellos, la diferencia de estos nmeros.
Podemos obtener que todos los nmeros en el tablero sean ceros?
Problema 10.
En el pas de Semabria viven 13 camaleones amarillos, 15 camaleones verdes
y 17 camaleones rojos. Cuando se encuentran dos camaleones de diferente color,
ellos adquieren simultneamente la coloracin del tercer color (por ejemplo, amar-
illo y verde se convierte en rojo). Puede resultar, que dentro de algn tiempo,
todos los camaleones tengan un solo color?
Indicacin.
Presten atencin, el problema 6 puede ser resuelto solo con la ayuda de las
consideraciones de paridad de los nmeros de casillas en la tabla a diferencia de
los problemas 7 y 8.
Analicemos la solucin del problema 10. En qu consiste la operacin de-
scrita? En que "se pierden" dos camaleones de dos colores diferentes y "aparecen"
dos camaleones de un tercer color. Si adivinamos que la magnitud-invariante se
necesita determinar por conjunto de nmeros (a, 6, c), donde a, 6 y c es la canti-
dad de camaleones amarillos, verdes y rojos respectivamente, entonces despus la
solucin se obtiene prcticamente enseguida. En realidad la operacin que est
descrita en la condicin, significa que del conjunto (a,6,c) se obtiene el conjunto
(a 1, 6 l,c + 2) o el conjunto (a l,6 + 2, c 1) o el conjunto (a + 2, 6 l,c 1)
-esto depende de la coloracin que adquieren los camaleones. Es claro, que las
diferencias entre los nmeros del conjunto o no cambian, o cambian en 3, lo que
significa que los residuos de estas diferencias al ser divididos por 3 no cambian,
ellos son invariantes. Pero inicialmente a b 13 15 = 2, y en el caso si todos
los camaleones son rojos, a 6 = 0 0 = 0. Los nmeros O y 2 tienen diferentes
residuos al ser divididos por 3, lo que demuestra la imposibilidad de tal situacin
en el pas. De forma anloga se analizan los casos cuando todos los camaleones
sean amarillos o todos sean verdes.
Para los profesores.
Si el tema sobre "Paridad" ya fue visto y ustedes resolvieron los problemas en
los cuales la paridad figuraba como invariante, es necesario que recuerden esto a
sus estudiantes.
El terna de "Invariantes" tiene un carcter bastante abstracto y hasta el mismo
principio del uso de las invariantes frecuentemente queda como no entendido y
algo difcil para los estudiantes. Por eso hay que prestar especial atencin para
que los estudiantes entiendan la lgica misma del uso de las invariantes. Es
necesario, sobre todo, analizar cuidadosamente las soluciones de los problemas
acerca del tema y del mismo modo tratar de lograr que cada uno de los estudiante
resuelva por s mismo, alguno de estos. D muchos ejemplos que ilustren las
soluciones de los problemas, intente hacer el resumen lo ms evidente posible, y
la lgica de la solucin lo ms clara posible. Como siempre, intente introducir la
palabra desconocida "invariante" y formule la filosofa general de las invariantes
solo despus de, que los estudiantes resuelvan o analizen varios problemas sencillos
en el uso de esta tcnica.
Claro que lo principal, en la solucin de los problemas sobre invariantes, es
que inventen una invariante por s mismos. Es esto un verdadero arte, el cual
es posible adquirir con la prctica en la solucin de problemas semejantes. Sin
embargo, no hay que olvidar lo siguiente:
a) Las magnitudes inventadas deben ser invariantes;
b) Esas invariantes deben dar diferentes valores para dos objetos dados en las
condiciones de loe problemas;
c) Es necesario, fijar enseguida la case de objetos para los cuales va a deter-
minarse nuestra magnitud.
Analicemos otro problema de ms importancia.
Problema 11.
En los vrtices de un polgono regular de 12-lados estn distribuidos los nmeros
+1 y 1 de tal forma, que en todos loe vrtices, menos en uno, estn los +1. Se
permite cambiar el signo de los nmeros en cualesquiera k vrtices consecutivos.
Es posible lograr con tales operaciones que el nico nmero 1 se desplaze al
vrtice vecino desde la posicin inicial, si a) k = 3; b) k = 4; c) k = 6?
Solucin.
La respuesta en todos los puntos ee negativa. La demostracin se hace por un
esquema nico: consideremos algn conjunto de vrtices, que posea la propiedad
de que cualquier conjunto de fc vrtices consecutivos contenga un nmero par de
vrtices denotados (ver la fig.1.3).
Verifique que esta propiedad se cumple para los conjuntos indicados en la
fig.1.3
Figure 1.3:
En calidad de inavariante consideremos el producto de todos los nmeros en
los vrtices indicados.
Para los profesores.
En esta solucin se encontr la idea difundida en el mtodo del invariante
escoger en cada objeto alguna parte, en la cual los cambios, provocados por las
operaciones resueltas, se ve particularmente sencillo.
Indicacin: con la ayuda de esta idea se resuelven los problemas 7 y 9.
. A propsito, intenten hacer la siguiente pregunta insidiosa a los estudiantes:
nosotros demostramos que -1 no puede desplazarse al vrtice vecino a la izquierda.
Se podr hacer de tal forma que -1 se desplaza al vrtice vecino a la c
1.2. 2. Coloracin
Un gran nmero de los problemas acerca del invariante se resuelven con la ayuda
de un invariante de tipo especfico llamado "coloracin". Aqu tenemos un ejemplc
tpico:
Problema 12.
La figura "camello" se desplaza por un tablero de dimensin 1
del tipo (1,3) (osea la figura se desplaza inicialmente a la casilla vecina, y despus
se desplaza tres casillas ms en sentido perpendicular; por ejemplo el cab,
se desplaza con jugada del tipo (1,2)). Se puede con la jugada del 'camel
desplazarse de alguna casilla inicial a una vecina?
\
,
Figure 1.4:
Solucin.
Respuesta: no se puede. Observemos la coloracin de un tablero de ajedrez
en colores blanco y negro. Entonces, que fcil es verificar, con cada jugada "el
camello" se desplaza de una casilla a otra del mismo color; dicho de otra forma:
el color de la carilla en la cual est "el camello" es el invariante. Pero como dos
casillas vecinas tienen diferente coloracin, entonces desplazarse de una casilla a
otra con la jugada del "camello" no es posible.
Aqu tenemos algunos problemas que se realizan por el mtodo de coloracin,
Problema 13.
a) Demostrar que un tablero de ajedrez de dimensin 8 X 8 no se puede cubrir
con 15 figuritas de dimensin 1 x 4 y con la figurita mostrada en la fig.2;l(l).
b) Demuestre que un tablero de dimensin 10 x 10 no se puede cubrir con las
figuritas mostradas en la fig.2.1(2).
c) Demuestre que al tablero de dimensin 102 X 102 no se puede cubrir con
figuritas de dimensin 1x4,
Indicacin para el problema 13b). Utilicen la coloracin de un tablero de
ajedrez.
Problema 14.
El fondo de una caja rectangular est cubierto por baldosas de dimensiones
1 x 4 y 2 x 2. Las baldosas fueron retiradas de la caja y una baldosa de 2 x 2 se
perdi. Esta baldosa fue reeemplazada por otra de dimensin 1x4. Demuestre
s
1
3
4
2
2
4
3
!
1

4
1
1
4
3
1
1
3
4
J
2
4
3
1
Figure 1.5:
que ahora el fondo de la caja no se puede cubrir.
Problema 15.
Es posible recorrer con el caballo un tablero de dimensiones 4 x N, habiendo
estado en cada casilla una vez exactamente y regresar a la casilla inicial?
Analicemos la solucin del problema 15. Coloreamos el tablero de 4 x N en 4
colores como se muestra en la fig.2.2. Supongamos que existe el desplazamiento del
caballo por las casillas del tablero dado. La coloracin dada posee tal propiedad
que si el caballo est en la casilla del color 1 (respectivamente 2), entonces con la
siguiente jugada l estar necesariamente en la casilla de color 3 (respectivamente
4). Del color 3 el juega al color 1 o 4, del color 4 el juega al color 2 o 3.
Por cuanto el nmero de casillas de colores 1 y 2 es igual al nmero de casillas
de colores 3 y 4, entonces en el caso del desplazamiento del caballo por las casillas
del tablero, loe colores de las parejas (1,2) y (3,4) se alternan . . . - (1,2) ->
(3
)
4) _ _ > (i
;
2) -* (3,4) > . . . Si aunque sea una vez el caballo juega del color 3
al 4 o del color 4 al 3, entonces obtenemos la sucesin . > (1,2) > (3,4) *
(3. 4) _ , . . .
) en
la
cu
al la alternacin se perturba. Esto significa que el nmero
de casillas de colores 1 o 2 es menor que el nmero de casillas de colores 3 o 4. Por
consiguiente, el caballo debe jugar obligatoriamente del color 3 al 1 y del color 4
al 2.Ahora es fcil ver que l puede estar slo en las casillas de los colores 1 y 3
(o 2 y 4). Pero esto es imposible, porque en este caso el caballo nunca estar en
las casillas de los colores 2 o 4 (1 y 3). Llegamos a una contradiccin.
Para los profesores.
1. Un poco de fantasa es suficiente para que uno mismo invente algunos
problemas sobre coloracin; revise por ejemplo todas las posibles variaciones de
figuritas y tableros en el problema 13. Recuerde que por lo comn la bsqueda de
la coloracin necesaria est orientada a la demostracin de la respuesta negativa.
2. Algo ms acerca del mtodo de la coloracin. Existen muchos problemas,
que se resuelven con la ayuda de la coloracin, los cuales a veces no tienen nada
que ver con el mtodo del invariante. Algunos variantes de este mtodo, por
completo, pueden ser considerados como temas independientes, a los cuales se les
puede dedicar una clase (una hora- hora y media) solucionando los problemas.
1.3. El residuo como un invariante
Antes de pasar a los siguientes problemas, veamos un poco de teoria.
Comenzemos con el Teorema sobre la divisin con residuo.
Teorema. Sea b un nmero positivo entero. Para cualquier nmero entero a
existe una nica pareja de nmeros enteros q y r tal, que
1) a = bq + r
2) o < r < b
Demostracin.
1. Consideremos la fraccin |. Cualquier nmero racional se encuentra entre
dos nmeros enteros vecinos
9<f ? +l.
Multipliquemos estas desigualdades por el nmero positivo b
bq < a < bq + b
Restamos bq
O < a - bq < b
Vj
Determinemos r = a bq.
Entonces obtenemos, que la condicin se cumple
1)
a
= bq + r
2) o < r < b.
Nosotros demostramos la existencia de la pareja de nmeros q y r, que satis-
facen la condicin del teorema.
2. Ahora demostremos su unicidad.
Supongamos, que existe una pareja ms q' y r' tal que
a =bq' + r'
Entonces bq + r =bq
1
+ r'
10
q') = r' r es divisible por b y satisface a las condiciones
6 < r' r < b.
Por eso r' ~ r = 0.
Esto significa que b(q~q') =0. Asi como b ^ O, entonces q q
1
0. Obtenemos
r' = r y q' = q.
El teorema est demostrado por completo.
Definicin. Al dividir a sobre se define r como el residuo y q como el cociente.
Congruencias
Sea mun nmero entero positivo, el cual ser fijo hasta el final del pargrafo.
Definicin. El nmero a se define congruente ai nmero 6 mdulo m, si a 6
es divisible por m.
Si a es congruente con b mdulo m, entonces esto se escribe asi:
a = 6(mod ni)
Esta anotacin se llama la congruencia mdulo m.
Las propiedades principales de las congruencias.
1) a = a(modm)
2) a =&(rnodm) = b = a(modro)
3) o =&(modm)&$ = c(modm) =$ a = c(modm)
4) Las congruencias se pueden sumar y multiplicar como las igualdades, osea
a =6(modm)&; c = d(modm) =*
i) a 4- c =b 4- (modm)
ii) ac s 6d(modm)
5) Cualquier nmero entero a es congruente mdulo m con su residuo al dividir
a sobre m.
7 ejemplos ms de problemas sobre el invariante. Estos problemas son curiosos
porque aqu el invariante sirve como residuo con respecto a algn mdulo. Esto
es un caso bastante difundido (ver los problemas 3,7-9 que son los residuos con
respecto al mdulo 2, el problema 11 con respecto al mdulo 3)
Problema 16.
Don Quijote tiene dos espadas mgicas,una de las cuales puede decapitar 21
cabezas al Dragn, y la segunda decapita 4 cabezas, pero entonces al Dragn le
crecen 1985 cabezas nuevas. Podr Don Quijote decapitar todas las cabezas al
Dragn, si inicialrnente ella tena 100 cabezas? (Observacin: si, por ejemplo
al Dragn le quedan slo tres cabezas, entonces no se pueden decapitarlas con
ninguna de las espadas).
Problema 17.
11
En los pases Di y Dali la unidad monetaria son los dileros y loe daleros
respectivamente, adems en Dili, un dilero se canjea por 10 daleros y en Dali,
el dalero se canjea por 10 dileros. Un financista principiante tiene 1 dilero, l
puede viajar libremente de un pas al otro y canjear su dinero en ambos pases.
Demuestre, que la cantidad de daleros que posee el financista nunca se podr
igualar con la cantidad de dileros.
Problema 18.
La mquina de cambio entrega por una moneda otras cinco. Se podr con la
ayuda de la mquina cambiar un rublo metlico por 26 monedas?
Problema 19.
Hay tres mquinas de escribir automticas. La primera mquina automtica
al leer la tarjeta con los nmeros a y 6 entrega otra con los nmeros a + 1 y
6 + 1; la segunda mquina al leer la tarjeta con los nmeros pares a y b entrega
una tarjeta con los nmeros a/2 y 6/2; la tercera mquina al leer una pareja de
tarjetas con los nmeros a, 6 y 6, c entrega una tarjeta con los nmeros a, c. Todas
las mquinas devuelven las tarjetas introducidas. Se podr de la tarjeta (5,19)
obtener la tarjeta (1,1988) con la ayuda de estas mquinas?
Problema 20.
En el tablero est escrito el nmero 8
n
. Se calcula la suma de las cifras de
este nmero, luego se calcula la suma de las cifras del nmero obtenido, y as
sucesivamente hasta que no resulte un nmero de 1 dgito. Qu nmero ser
ste, si n = 1989?
Problema 21.
En un tubo de ensayo hay tres tipos de amebas marcianas: A, B y C. Dos
amebas de dos tipos diferentes se funden en una ameba del tercer tipo. Despus
de algunas fusiones de las amebas en el tubo result slo una ameba. Cul es su
tipo, si la cantidad inicial de amebas de tipo A era 20, de tipo B 21 y del tipo C
22?
Problema 22.
Una ficha se desplaza por un tablero cuadrado, en cada jugada se mueve o
a una casilla arriba, o a una casilla a la derecha, o por la diagonal abajo y a la
izquierda (ver fig.3.1) Puede la ficha recorrer todo el tablero habiendo estado en
todas las casillas exactamente una vez y terminar en la casilla vecina a la derecha
de la inicial?
Intentemos modelar el proceso de solucin del problema 10.
Para los profesores.
Si ustedes exponen la solucin a los estudiantes, entonces intenten mostrarla
12
Figure 1.6:
en forma de cuento en donde explique, como es el procedimiento, como adivin
que el problema es sobre el invariante, etc...
As, la forma exterior del problema: dado un conjunto de operaciones resueltas
y nos piden aclarar si se puede obtener de una tarjeta otra, lo que impulsa a buscar
necesesariamente el invariante. Empezemos con la bsqueda.
Primera operacin: (a, b) -> (a + I , b + 1). Qu no cambia en esta operacin?
Claro que la diferencia de los nmeros en la tarjeta: (a+1) (& +1) = a b. Pero
la segunda operacin cambia la diferencia: a/2 6/2 (a b)/2, la divide por la
mitad. La tercera operacin adiciona estas diferencias: a c = (a b) + (b c).
Todo esto nos hace pensar, que al parecer el invariante no es la diferencia
misma, sino... que entonces? Qu se puede hacer con este nmero? Segu-
ramente que la respuesta a esta pregunta enseguida no nos viene a la cabeza.
Vamos a empezar con una pequea investigacin. Intentemos al azar obtener
algunas tarjetas de la dada:
(5,19)-* (6, 20) (1)
(6,20) -* (3,10) (2)
(3,10) -> (20,27) (3)
13
(6,20), (20,27)-,(6,27) (4)
Por es suficiente. Ahora si podemos ver nuestros frutos. Tenemos un conjunto
de tarjetas: (5,19), (6, 20), (3,10), (20,27), (6,27). Calculemos para ellos la difer-
encia de los nmeros en la tarjeta y obtenemos el siguiente cojunto de nmeros:
14,14,7,7,21. Claro que aqu enseguida adivinamos lo que tenamos que de-
mostrar! Por supuesto que nuestra diferencia a b siempre ser divisible por 7.
Esto es fcil de demostrar, slo hay que volver a mirar que sucede con la difrencia
para las operaciones resueltas (ver arriba). Pero la tarjeta que queremos obtener
(1,1988) la diferencia de los nmeros es igual a 1 1988 =1987 y no es divisible
por 7, El problema est resuelto.
Una serie de problemas finales:
Problema 23.
En una tabla de dimensin m x n los nmeros estn distribuidos de tal forma
que la suma de los nmeros en cualquier fila y columna es igual a 1. Demostrar
que m = n.
Observacin.
Por extrao que paresca, pero en algn sentido este tambin es un problema
sobre el invariante.
Problema 24.
En la mesa hay 7 vasos, todos boca abajo. Se permite en una jugada voltear
4 cualesquiera vasos. Es posible colocar en forma correcta todos los vasos con
varias jugadas?
Problema 25.
En los vrtices de un cubo estn distribuidos unos nmeros: 7 ceros y un uno.
Est permitido, en un paso, aumentar una unidad a los nmeros en los extremos
de cualquier arista del cubo. Podemos lograr que todos los nmeros sean iguales?
Y es posible lograr que todos los nmeros sean divisibles por 3?
Problema 26.
Un crculo est dividido en 6 sectores, como en el problema 3, y en ellos estn
distribuidos, en el sentido de las manecillas del reloj, los siguientes nmeros:
1,0,1,0,0,0. Est permitido aumentar una unidad a los nmeros de dos cua-
lesquiera sectores vecinos. Es posible lograr que todos los nmeros en los sectores
sean iguales con tales operaciones?
Problema 27.
14
En el problema 20 aclare que tarjetas se pueden obtener de la tarjeta (5,19),
y cuales no.
Problema 28.
En la mesa hay montones de 1001 piedras. La jugada consiste en que de
cualquier montn que contenga ms de una piedra, se bota una piedra, y despus
uno de los montones es dividido en dos. Pueden quedar en la mesa slo montones
que contengan tres piedras en varias jugadas?
Problema 29.
En serie estn escritos los nmeros 1,2,3,.. .,n. En un paso se permite cambiar
de lugar cualesquiera dos nmeros. Es posible obtener el orden inicial de los
nmeros, despus de 1989 de tales operaciones?
Problema 30.
Dado un trio de algunos nmeros. Con dos cualesquiera nmeros se permite
hacer lo siguiente: si estos nmeros son iguales a a y 6, entonces los podemos
cambiar por (o 4- b)/\/2 y (a b)/V2. Se puede obtener con la ayuda de tales
operaciones obtener un trio (1, \/2,1 4- \/2) de un trio (2, \/2, l/\/2)?
Estos ejercicios son ms o menos difciles que la mayora de los problemas 1-23,
pero pueden ser buenos problemas para investigar en la casa.
Para los profesores.
1. Los problemas sobre el invariante son bastante conocidos. As por ejemplo,
en la olimpiada de San Petersburgo cada ao, por lo menos dos de tres problemas
se resuelven por el mtodo del invariante. Esto se debe comunicar a los estudi-
antes, pues es posible que esto los impulse a una aprendizaje ms detallado sobre
el tema (aunque sea por razones de carcter deportivo).
2. El concepto de invariante se utiliza en las ms diferentes ramas de la
ciencia. Si sus estudiantes conocen lo suficiente sobre fsica, entonces en calidad
de ejemplo, usted con sus alumnos pueden analizar las diferentes consecuencias
de la ley de conservacin de la energa, igualmente los teoremas del siguiente tipo:
ley de conservacin del impulso, etc.
3. Los estudiantes deben entender, que si el invariante escogido da significados
iguales para dos objetos dados, entonces esto no significa que ellos se pueden
obtener uno del otro con la ayuda de las operaciones dadas en el problema. Esto es
una equivocacin bastante estndar, la cual aparece generalmente con los primeros
conocimientos del mtodo del invariante. Invente durante la clase varios problemas
sencillos que desmientan esta "idea".
4. Recordemos una vez ms algunos invariantes estndar:
1) el residuo con respecto a algn mdulo, problemas 3,7-11,17-22.
15
1
2
I
2
1
4
3
4
3
4
1
2
1
2
1
4
3
4
3
4
1
2
1
2
1
Figure 1.7:
2) Sealar las partee del objeto, problemas 7,9,12.
3) Coloracin, problemas 13-16.
4) La expresin algebraica de los problemas dados; como en los problemas
4,26,30.
1.4. Soluciones
5. En calidad de invariante consideren la siguiente magnitud: Sea que cada loro
tenga el nmero igual al nmero de su palmera contando desde la izquierda.
Entonces la suma de los nmeros de loros S es el invariante.
7. Demuestren que la paridad del nmero de casillas negras entre las cuatro
de los rincones no cambia cuando se vuelven a pintar.
8. El problema se resuelve como el problema 7. Necesita solo considerar las
otras cuatro casillas siguientes: al, a2,61,62 ( en las notaciones de ajedrez).
9. Verifique que la paridad de los nmeros en el tablero no cambia.
13. a) Utilize la coloracin de la tabla en dos colores de un color en cada fila
en forma alternada, c) Utilize la misma coloracin en cuatro colores.
14. Consideremos la coloracin en cuatro colores, mostrada en la fig.3.2.
Entonces cada baldosa de 2 X 2 contiene exactamente una casilla de color 1, y
cada baldosa de 1 x 4 contiene o ninguna casilla de color 1, o 2 casillas de color 1.
Por lo tanto, la paridad del nmero de baldosas de 2 x 2 deber coincidir con
la paridad del nmero de casillas de color 1, lo que demuestra la afirmacin del
16
problema.
16. El invariante es el residuo del nmero de las cabezas del dragn con
respecto al mdulo 7.
17. El invariante es el residuo con respecto al mdulo 11 de la diferencia entre
los nmeros de dileros y daleros que posee el financista.
18. No se puede. Observe los residuos con respecto al mdulo 4.
20. Utilize la siguiente afirmacin: la suma de las cifras de un nmero y el
mismo nmero tienen los residuos iguales con respecto al mdulo 9.
21. El tipo de la ameba es B. Observe la paridad de las siguientes diferencias
N(A) - N(B),N(B) - N(C),N(C) - N(A), donde N(X) es el nmero de las
amebas del tipo X.
22. La suma de los nmeros de la fila y de la columna en cada paso o
disminuye en 2, o aumenta en 1. Esto significa que su residuo con respecto al
mdulo 3 aumenta cada vez en 1. Como el nmero total de pasos es n
2
- 1, y
al final la suma deber ser en un uno ms que la inicial, entonces obtenemos que
n
2
- 2 deber ser divisible por 3, lo que es imposible. Por consiguiente no hay tal
recorrido. Observacin: Preste atencin a lo siguiente: la solucin est dada sin
la mencin de la palabra "invariante". Qu es lo invariante aqu?
23. La suma de los nmeros en la tabla no depende del mtodo de su clculo.
Precisamente, en ese sentido, este es un problema sobre el invariante.
24-26. No se puede. En el problema 24 como invariante sirve la paridad del
nmero de vasos que estn en posicin incorrecta; en el problema 25 es necesario
denotar cuatro vrtices del cubo tales, que ningunos dos de ellos no estn unidos
por la arista, despus de esto considerar la diferencia entre la suma de los nmeros
en los vrtices marcados y la suma de los nmeros en los vrtices no marcados;
en el problema 26 es necesario enumerar los sectores con nmeros desde 1 hasta
6 en orden, despus de esto considerar la diferencia entre la suma de los nmeros
en los sectores "pares" y la suma de los nmeros en los sectores "impares".
27. Exactamente aquellas tarjetas (a, &), para las cuales a < 6 y 6 - a e s
divisible por 7.
28. No se puede. Consideren la magnitud 5 que ee igual a la suma de la
cantidad de piedras y de montones.
29. No, no puede. En calidad de invariante considere la paridad de la magni-
tud p, igual al nmero de parejas (a, 6), en las cuales el nmero a est a la derecha
del nmero b, y adems a > b,
30. No se puede. Consideren la suma de los cuadrados de los nmeros del
trio.
17

You might also like